LSAT and Law School Admissions Forum

Get expert LSAT preparation and law school admissions advice from PowerScore Test Preparation.

 studyhelp20
  • Posts: 28
  • Joined: Dec 09, 2020
|
#82130
Can you please explain the correct answer for this question and explanations of the incorrect answers? thanks
 Adam Tyson
PowerScore Staff
  • PowerScore Staff
  • Posts: 5153
  • Joined: Apr 14, 2011
|
#82357
Happy to, studyhelp20!

If H and K are to be on the same day, we should give some thought to what day that could be, and an important part of that process will be thinking about the JG block and the final rule that keeps K and G fairly close to each other. It's the interaction of those rules that drives this question.

If K and H were on Wednesday, then J would be Thursday and G would be Friday. This puts K and G too far apart, even if you put G in the morning on Friday. Try it and you'll see that J and L are both after K and before G, which is one too many according to that last rule.

Putting K and H on Thursday prevents us from putting the JG block anywhere, so that's out.

So H and K must be on Friday, and because of the first rule H is in the morning and K must be in the afternoon. That puts J sometime on Wednesday and J sometime on Thursday. Note that G doesn't have to be in the afternoon in order to comply with the last rule (although it could be). G could be in the morning on Thursday, with no client meeting (our X in the main diagram) that afternoon, placing L on Wednesday morning and J on Wednesday afternoon. In that case, there would still be only one client meeting - H - between G and K. Or, flip the G and X on Thursday and it's still fine.

In the alternative, G could be Thursday afternoon with L in the morning that day, and then J can be in either position on Wednesday (X, the empty slot, being in the other Wednesday position). That's where the correct answer - C - comes from. None of the others are possible:

A. G cannot be Friday because H and K fill those two slots
B. H cannot be Wednesday, as discussed above (this breaks the rule about K and G)
D. K can't be Thursday because it must be Friday (this would break the JG block)
E. L can't be Friday because H and K fill those two slots (again, see above for why the H and K combo can only happen on Friday)

These two diagrams might help you see it:

L G/X H
J X/G K

J/X L H
X/J G K

Get the most out of your LSAT Prep Plus subscription.

Analyze and track your performance with our Testing and Analytics Package.